Stron induction, multiple choice not sure how he got his answer

In summary, the person is trying to figure out how to test if c and d are correct. They are not sure where to start, and need help figuring out how to approach the problem.
  • #1
mr_coffee
1,629
1
Hello everyone, I'm confused on what the answer is here, it looks like he circled every answer but i don't see how that's possible. So maybe he cirlced c and b and marked a and b wrong. http://suprfile.com/src/1/3pyqpub/lastscan.jpg How would I test to verify that c and d are correct?
if b1 = 3, b2 = 6 and b_n = b_n-1 + b_n=2
for all integgers n >= 3.

n has to be >= 1, so do I plug in
n = 1, n = 2, and n = 3?

like n = 1, so b_n = b_0 + b_n-1

But b_0 and b_n-1 isn't even listed so I am not sure where to go from this.
 
Last edited by a moderator:
Physics news on Phys.org
  • #2
Yes, all four of those are correct. (You would, of course, have to verify that b1 is divisible by 3 and, for b, that b2 is divisible by 3, for a and c, that both b2 and b3 are divisible by 3.

like n = 1, so b_n = b_0 + b_n-1

But b_0 and b_n-1 isn't even listed so I am not sure where to go from this.
There is no b_0: the sequence starts with b_1. The first one you would have to calculate is b_3= b_1+ b_2= 3+ 6= 9 which is divisible by 3.

It really doesn't matter where you start the "induction step", as long as you have verified the statement separately for each n less than that.

Of course, if it is true that "ai is divisible by 3 for all [itex]i\le k[/itex]" then we can write ak= 3m and ak-1= 3n for some integers m and n. Then ak+1= ak+ ak-1= 3n+ 3m= 3(n+m), a multiple of 3.
 
  • #3
Thanks Ivy, i'll take another look at it!
 
  • #4
c and d are correct, a and b are not correct.
The problem with a is that you start the induction on k > 3. It doesn't check to see if it's true for k = 3.
The problem with b is that you assume the statement is true for all i <= k. So you assume it's true when i = k and then you show that it's true for k--circular logic.
 
  • #5
THanks for the responce!
i'm confused on how you figure out ur boundaries...
how do u know which boundary is correct or incorrect for i and also how did u know what k to test for? I was able to figure out easy problems using induction but I'm quite lost when it comes to strong induction on how you set up your base case and step. any guidance would be great!
 

Related to Stron induction, multiple choice not sure how he got his answer

What is strong induction?

Strong induction is a method of mathematical proof that is used to establish a statement for all positive integers. It involves using the truth of all previous cases to prove the truth of the current case.

How does strong induction differ from regular induction?

In regular induction, the truth of the next case is established by the truth of the previous case. In strong induction, the truth of the next case is established by the truth of ALL previous cases.

Why is strong induction useful?

Strong induction is useful because it allows us to prove statements that cannot be proven using regular induction. It also allows us to prove statements about sequences and series.

What is the role of the base case in strong induction?

The base case in strong induction is used to establish the truth of the first case. Without a true base case, the proof will not hold for all cases.

Can strong induction be used to prove all mathematical statements?

No, strong induction can only be used to prove statements that have a well-defined ordering, such as the set of positive integers. It cannot be used for other types of statements, such as statements about real numbers.

Similar threads

  • MATLAB, Maple, Mathematica, LaTeX
Replies
1
Views
1K
  • Set Theory, Logic, Probability, Statistics
Replies
2
Views
1K
  • Calculus and Beyond Homework Help
Replies
2
Views
1K
  • Calculus and Beyond Homework Help
Replies
5
Views
1K
  • Quantum Physics
Replies
5
Views
883
  • Calculus and Beyond Homework Help
Replies
4
Views
3K
  • Introductory Physics Homework Help
Replies
3
Views
339
  • Calculus and Beyond Homework Help
Replies
2
Views
1K
  • Introductory Physics Homework Help
Replies
5
Views
1K
  • Introductory Physics Homework Help
Replies
8
Views
1K
Back
Top